100 students are interviewed to see which of biology, chemistry or physics they prefer. 42 of the students are girls. 5 of the girls like biology best. 4 of the boys prefer physics. 13 out of the 21 who prefer chemistry are girls. What percentage of the students prefer biology?

Answers

Answer 1

Answer:

A total of 51 people prefer biology, 51%

Step-by-step explanation:

Girls: 42

Boys: 58

Biology: 5 girls and 46 boys(100-42=58, 58-[4+8]=46)

Physics: 4 boys and 24 girls(42-[5+13])

Chemistry: 13 girls and 8 boys(21-13=8)

I think this is the answer...


Related Questions

Solve the equation and enter the value of x below. 7(x + 9) + 5 = 96​

Answers

Hello!

[tex]\large\boxed{x = 4}[/tex]

7(x + 9) + 5 = 96

Distribute:

7x + 63 + 5 = 96

Combine like terms:

7x + 68 = 96

Subtract 68 from both sides:

7x = 28

Divide both sides by 7:

x = 4

Answer:

[tex]\fbox{x = 7}[/tex]

Step-by-step explanation:

7(x + 9) + 5 = 96

Solve for x.

7(x + 9 ) + 5 = 96

Step 1 :- Distribute 7.

7 × x + 7 × 9 + 5 = 967x + 63 + 5 = 96

Step 2:- Add 63 and 5.

7x + 68 = 96

Step 3 :- Move constant to the right-hand side and change their sign.

7x = 96 - 68

Step 4 :- Subtract 68 from 96.

7x = 28

Step 5 :- Divide both side by 7.

[tex]\frac{7x}{7} \\ [/tex] = [tex]\frac{ 28} {7}\\[/tex] x = 4

Marcella incarica Dora di aiutarla a suddividere 35 perle tra le sue tre figlie. Alla figlia maggiore vuole regalare la metà, alla mediana la terza parte e alla minore la nona parte. Inoltre Marcella vuole dare una perla a Dora come ricompensa. Quante perle riceverà ciascuna figlia?

Answers

Answer:

La hija mayor recibirá 17.5 perlas, la hija del medio recibirá 11.66 perlas, y la hija menor recibirá 3.88 perlas.

Step-by-step explanation:

Dado que Marcella le pide a Dora que la ayude a dividir 35 perlas entre sus tres hijas, y quiere darle la mitad a la hija mayor, la tercera parte a la del medio y la novena a la menor y, además, Marcella quiere darle una perla a Dora como recompensa, para determinar cuántas perlas recibirá cada hija se debe realizar el siguiente cálculo:

35 / 2 = Mayor = 17.5 perlas

35 / 3 = Medio = 11.66 perlas

35 / 9 = Menor = 3.88 perlas

Así, la hija mayor recibirá 17.5 perlas, la hija del medio recibirá 11.66 perlas, y la hija menor recibirá 3.88 perlas.

4.5hour into second

Answers

Step-by-step explanation:

4 .5 ×60min

4.5×60×60sec

16200 sec

1 hour -3600 sec                                                                                                    4,5 hoer - x sec                                                                                                         x=4,5* 3600=45*360=16200 sec                                                                     Answer: 4,5 hour = 16200 sec

A new school has x day students and y boarding students.
The fees for a day student are $600 a term.
The fees for a boarding student are $1200 a term.
The school needs at least $720 000 a term.
Show that this information can be written as x + 2y ≥ 1200.​

Answers

Given:

The fees for a day student are $600 a term.

The fees for a boarding student are $1200 a term.

The school needs at least $720000 a term.

To show:

That the given information can be written as [tex]x + 2y\geq 1200[/tex].​

Solution:

Let x be the number of day students and y be the number of boarding students.

The fees for a day student are [tex]\$600[/tex] a term.

So, the fees for [tex]x[/tex] day students are [tex]\$600x[/tex] a term.

The fees for a boarding student are [tex]\$1200[/tex] a term.

The fees for [tex]y[/tex] boarding student are [tex]\$1200y[/tex] a term.

Total fees for [tex]x[/tex] day students and [tex]y[/tex] boarding student is:

[tex]\text{Total fees}=600x+1200y[/tex]

The school needs at least $720000 a term. It means, total fees must be greater than or equal to $720000.

[tex]600x+1200y\geq 720000[/tex]

[tex]600(x+2y)\geq 720000[/tex]

Divide both sides by 600.

[tex]\dfrac{600(x+2y)}{600}\geq \dfrac{720000}{600}[/tex]

[tex]x+2y\geq 1200[/tex]

Hence proved.

If you know the answer to any of these please Ill mark brainliest.

Answers

Answer:

Step-by-step explanation:

1) 2/9 = 185/x

  2x = 1665

x = 832.5 or 8.325 meters

2) unit rate is the cost of one unit

i.e a dozen eggs cost $1.20 that

would be 10 cents each

3) 300 tissues for $3.75 vs. 250 for $2.99

one is 1.24 cents per tissue

the other 1.19 ... the 250 package is a better buy

A 4-column table with 4 rows. Column 1 is labeled number of friends with entries 3, 5, 7, 9. Column 2 is labeled Carnival Cost with entries 51.5, 75.5, 99.5, 123.5. Column 3 is labeled Aquarium Cost with entries 43.5, 72.5, 101.5, 130.5. Column 4 is labeled Wave Pool Cost with entries 50.25, 61.25, 85.75, 110.25.
Gale wants to compare the cost of the events.

Aquarium: $14.50 each ticket

Carnival: c = 15.5 + 12f

Wave pool: $16.75 each, but $12.25 each for groups larger than 4

Which of the claims Gale makes is true?
The table represents all possibilities.
The carnival always costs the least.
The Aquarium always costs the most.
(7, 101.5) is an ordered pair used to represent the aquarium cost.

Answers

Answer:

D.(7, 101.5) is an ordered pair used to represent the aquarium cost.

Hope it helps :)

Step-by-step explanation:

Answer: it is d - (7, 101.5) is an ordered pair used to represent the aquarium cost

Step-by-step explanation: got it right on edge 2022

f(b) = 7b^3 +8b and g(b) = b ^2+ b - 10. What is f(b)-g(b)?

Answers

Answer:

[tex]{ \bf{f(b) - g(b) : }} \\ { \tt{ = ( {7b}^{3} + 8b) - ( {b}^{2} + b - 10)}} \\ = ( {7b}^{3} - {b}^{2} + 7b + 10)[/tex]

Answer:

[tex]f(b) - g(b) = 7b^3 -b^2 + 7b +10[/tex]

Step-by-step explanation:

[tex]f(b) = 7b^3 + 8b \\\\g(b) = b^2 + b - 10\\\\f(b) - g(b) = (7b^3 + 8b ) - ( b^2 + b -10)[/tex]

               [tex]= 7b^3 + 8b - b^2 - b + 10\\\\=7b^3 - b^2 +7b + 10[/tex]

Which of the following is equivalent to 2/x + 3/x−1 for x>1 ?

pls explain

Answers

Answer:

Step-by-step explanation:

I'm confused, can someone please help?!​

Answers

Answer:

B

Step-by-step explanation:

Pls brain list if right :>

Answer:

If ABCD is congruent to RSTU

AB≅RS

BC≅ST

CD≅TU

AD≅RU

and ∠A≅CR

∠B≅∠S

∠C≅T

∠D≅CU

ANSWER: ∠A≅∠U

------------------------------

hope it helps...

have a great day!!

Selecting a few households from New York City and observing whether or not they own stocks when it is known that 28% of all households in New York City own stocks. Is this experiment a binomial experiment? Explain why.

Answers

Answer:

Since for each household there are only two possible outcomes, the a household owning stock is independent of any other household and there is a fixed number of trials, this experiment is a binomial experiment.

Step-by-step explanation:

We have to take into consideration three things:

For each household, there are only two possible outcomes, either they own stocks, or they do not.

The probability of a household owning stock is independent of any other household, that is, for any household, the probability of someone owning stocks is 28%.

There is a fixed number of trials.

Is this experiment a binomial experiment?

Since for each household there are only two possible outcomes, the a household owning stock is independent of any other household and there is a fixed number of trials, this experiment is a binomial experiment.

rationalization
[tex] \frac{6}{ \sqrt{3} } [/tex]

Answers

Answer:

2√3

Step-by-step explanation:

To rationalize, we multiply the denominator and numerator by the surd at the base

We have it that;

6/√3 * 1

= 6/√3 * √3/√3

= 6 √3/3 = 2√(3

Solve the equation: 12 - x (x - 3) = (6 - x)(x + 2)

Answers

Answer: the answer is 0

Why 0?:

Step 1: Simplify both sides of the equation

-x * x= -x²

-x * -3= 3x all  together it is ( -x²+3x+12)

for the other side:

-x * x= -x²

6 * x= 6x and x * 2= 2x and 6 * 2= 12

then subtract 6x-4x= 2x so final would be = −x²+4x+12

Step 2: Add x² to both sides.

−x²+3x+12+x²=−x²+4x+12+x² then we are left with 3x+12=4x+12

Step 3: Subtract 4x from both sides.

3x+12−4x=4x+12−4x  --> −x+12=12

Step 4: Subtract 12 from both sides.

−x+12−12=12−12

−x=0

Step 5: Divide both sides by -1 or -x

x=0

Please help me out here

Answers

Answer:

vbw-kafw-hxy p.l.e.a.s.e join

Answer:

146 cm²

Step-by-step explanation:

The net is composed of 3 sets of congruent rectangles

top/ bottom + front/ back + sides

SA = 2(9 × 5) + 2(9 × 2) + 2(5 × 2)

     = 2(45) + 2(18) + 2(10)

     = 90 + 36 + 20

     = 146 cm²

f(x) = 3x + 2
What is (5)?
O A. 21
B. 17
C. 15
O D. 10


Answers

B. 17

X=5

f(5) = 3x + 2
= 3(5) + 2
= 15+2
= 17

Hope this help you
Answer is 17...............

How do I answer number 1

Answers

Answer:

#1 Haley is correct and Lacey is incorrect

#2 Kenji is incorrect.

Step-by-step explanation:

#1. x^3 (x^2) = x^5 but this same law doesn't apply to addition of numbers with exponents.

#2 The law of exponents doesn't apply to numbers with different bases that are not multiples of each other such as 3 and 4, so Kenji's simplification is not correct.

helppp!! I NEED HELP PLEASE

Answers

Given:

The table of values for the function f(x).

To find:

The values [tex]f^{-1}(f(3.14))[/tex] and [tex]f(f(-7))[/tex].

Solution:

From the given table, it is clear that the function f(x) is defined as:

[tex]f(x)=\{(-14,11),(-7,-12),(-12,-5),(9,1),(10,-2),(-2,13)\}[/tex]

We know that if (a,b) is in the function f(x), then (b,a) must be in the function [tex]f^{-1}(x)[/tex]. So, the inverse function is defined as:

[tex]f^{-1}(x)=\{(11,-14),(-12,-7),(-5,-12),(1,9),(-2,10),(13,-2)\}[/tex]

And,

[tex]f^{-1}(f(a))=f^{1}(b)[/tex]

[tex]f^{-1}(f(a))=a[/tex]              ...(i)

Using (i), we get

[tex]f^{-1}(f(3.14))=3.14[/tex]

Now,

[tex]f(f(-7))=f(-12)[/tex]

[tex]f(f(-7))=5[/tex]

Therefore, the required values are [tex]f^{-1}(f(3.14))=3.14[/tex] and [tex]f(f(-7))=5[/tex].

Sam has 24 ball he gives 1/3 to Rita. How many ball did he give to Rita?​

Answers

Answer:

8

Step-by-step explanation:

Find 1/3 of 24, which is 8.

Answer:

Sam gave 8 balls to Rita.

Step-by-step explanation:

24 / 3 = 8

1/3 of 24 is 8, therefore Sam gave 8 balls to Rita.

Simplificar 2/3+1/5+2/4

Answers

Answer:

41/30

Step-by-step explanation:

2/3 + 1/5 + 2/4

= 41/30

Find the coordinates of the
midpoint M. *
A(-4,-8) and B(-1,4)
Please provide an explanation

Answers

Answer:

M(-5/2, -2)

Step-by-step explanation:

Add the x-coordinates & divide by 2.

Add the y-coordinates and divide by 2.

x-coordinates: -4, -1

(-4 + (-1))/2 = -5/2

y-coordinates: -8, 4

(-8 + 4)/2 = -4/2 = -2

M(-5/2, -2)

M(-5/2,-2) there u go that should be it


The line CD is defined by the points C(-2,1) and D(10,7).
Find the equation of the line CD.

Answers

Answer:

The equation of the line is; y = 0.5·x + 2

Step-by-step explanation:

The points that define the line CD = C(-2, 1) and D(10, 7)

The equation of the line can be presented in the form of the general equation of a straight line, y = m·x + c

Where;

m = The slope of the line = [tex]\dfrac{7 - 1}{10 - (-2)} = \dfrac{1}{2} = 0.5[/tex]

c = The y-intercept

From the obtained slope, m = 0.5, using point D(10, 7), the equation of the line in point and slope form is therefore;

y - 7 = 0.5·(x - 10)

From the above equation of the line in point and slope form, we get the general form of the equation of the line as follows

y - 7 = 0.5·(x - 10) = 0.5·x - 5

y - 7 = 0.5·x - 5

y = 0.5·x - 5 + 7 = 0.5·x + 2

y = 0.5·x + 2

The equation of the straight line in general is y = 0.5·x + 2.

pls help w explanation!!!

At her gym, Ximena spends 30 minutes on each aerobic workout and 20 minutes on each weight-lifting workout. Last week, Ximena spent between 190 and 230 minutes, inclusive, on 3 aerobic
workouts and w weight-lifting workouts. What is
one possible value of w?

Answers

Answer:

The possible values of W are 5, 6, and 7.

Step-by-step explanation:

Since at her gym, Ximena spends 30 minutes on each aerobic workout and 20 minutes on each weight-lifting workout, and last week, Ximena spent between 190 and 230 minutes, inclusive, on 3 aerobics workouts and W weight-lifting workouts, to determine what is one possible value of W the following calculation must be performed:

190 - (3 x 30) = X

190 - 90 = X

100 = X

100/2 = 5

140/2 = 7

Therefore, the possible values of W are 5, 6, and 7.

Plsss can someone answer the bottom question

Answers

Answer:

30

Step-by-step explanation:

[tex]\frac{75}{100} = \frac{x}{40}\\\\Cross- multiply:\\\\\\75*40=100x[/tex]

3000 = 100x

x = 30

Another way:

75/100 is in reality 3/4

and 3/4 of 40 = 30

40/4 = 10

10 * 3 = 30

If my answer is incorrect, pls correct me!

If you like my answer and explanation, mark me as brainliest!

-Chetan K

The answer is 30

Because its looking for 75% of 40, you have to turn 75% into a decimal by moving the placement by two.
75 turn into .75
After you do that you multiply 40 by .75 and you get 30, which is your answer. :)

Plss Answer!!!!!!!!!​

Answers

Answer:

Ecosystem: Trees

Ways to protect are as follows:-

Control over Forest FireDon't waste paper. Plant a tree

Come get your point with me :)

Answers

Answer:

IK≅WY

Step-by-step explanation:

What number should be subtracted from -3/4 to get 5/6?

Answers

Answer:

Let that rational number to be subtracted be x.Given,-5/​6 - x = 4/9= - x = 4/9+5/6= - x = 23/18x = - 23/18.

Step-by-step explanation:

If g(x) = 2 |x| − 1, what is g(−2.3)?

Answers

Answer:

g(-2.3) = 3.6

General Formulas and Concepts:

Pre-Algebra

Order of Operations: BPEMDAS

Brackets Parenthesis Exponents Multiplication Division Addition Subtraction Left to Right

Algebra I

FunctionsFunction Notation

Step-by-step explanation:

Step 1: Define

Identify

g(x) = 2|x| - 1

Step 2: Evaluate

Substitute in x [Function g(x)]:                                                                         g(-2.3) = 2|-2.3| - 1Absolute values:                                                                                               g(-2.3) = 2(2.3) - 1Multiply:                                                                                                             g(-2.3) = 4.6 - 1Subtract:                                                                                                            g(-2.3) = 3.6
3.6
2 *| -2.3 | - 1 = 3.6

The perimeter of a rectangular swimming pool is 56 meters. The width is 4 meters less than the length. What is the width of the swimming pool? 4 meters 4 meters 8 meters 8 meters 12 meters 12 meters 24 meters

Answers

Answer:

Length = 16 meters

Width = 12 meters

Step-by-step explanation:

Perimeter of a rectangle = 2(length + width)

Let

length = x

Width = (x - 4) meters

Perimeter of the rectangular pool = 56 meters

Perimeter of a rectangle = 2(length + width)

56 = 2{x + (x - 4)}

56 = 2(x + x - 4)

56 = 2(2x - 4)

56 = 4x - 8

56 + 8 = 4x

64 = 4x

x = 64/4

x = 16

length = x = 16 meters

Width = (x - 4) meters

= 16 - 4

= 12 meters

Solve an equation to find the missing angle

Answers

11.

[tex]6x = 30 \\ x = \frac{30}{6} \\ x = 5[/tex]

Missing angle:

[tex]6x \\ = 6 \times 5 \\ = 30[/tex]

_________________________________________

12.

[tex](4 + 5x) + (x + 2) = 180 \\ 6x + 6 = 180 \\ 6x = 180 - 6 \\ 6x = 174 \\ x = \frac{174}{6} \\ x = 29[/tex]

Missing angle 1:

[tex](4 + 5x) \\ = 4 + (5 \times 29) \\ = 4 + 145 \\ = 149[/tex]

Missing angle 2:

[tex]x + 2 \\ = 29 + 2 \\ = 31[/tex]

_________________________________________

13.

[tex]5x + (3x + 12) = 180 \\ 8x + 12 = 180 \\ 8x = 180 - 12 \\ 8x = 168 \\ x = \frac{168}{8} \\ x = 21[/tex]

Missing angle 1:

[tex]5x \\ = 5 \times 21 \\ = 105[/tex]

Missing angle 2:

[tex](3x + 12) \\ = (3 \times 21) + 12 \\ = 63 + 12 \\ = 75[/tex]

_________________________________________

14.

[tex]32 + (6x + 4) = 90 \\ 36 + 6x = 90 \\ 6x = 90 - 36 \\ 6x = 54 \\ x = \frac{54}{6} \\ x = 9[/tex]

Missing angle:

[tex](6x + 4) \\ = (6 \times 9) + 4 \\ = 54 + 4 \\ = 58[/tex]

_________________________________________

15.

[tex](2x + 1) + (x + 2) = 90 \\ 3x + 3 = 90 \\ 3x = 90 - 3 \\ 3x = 87 \\ x = \frac{87}{3} \\ x = 29[/tex]

Missing angle 1:

[tex](2x + 1 ) \\ = ( 2 \times 29) + 1 \\ = 58 + 1 \\ = 59[/tex]

Missing angle 2:

[tex]x + 2 \\ = 29 + 2 \\ = 31[/tex]

_________________________________________

16.

[tex](3x + 1) + (4 + 2x) = 90 \\ 5x + 5 = 90 \\ 5x = 90 - 5 \\ 5x = 85 \\ x = \frac{85}{5} \\ x = 17[/tex]

Missing angle 1:

[tex](3x + 1) \\ = (3 \times 17) + 1 \\ = 51 + 1 \\ = 52[/tex]

Missing angle 2:

[tex](4 + 2x) \\ = 4 + (2 \times 17) \\ = 4 + 34 \\ = 38[/tex]

What is true about an equation with infinite solutions?
When both sides of the equation are simplified, the coefficients are the same.
When both sides of the equation are simplified, the constants are different.
There are no input values that will result in a true statement.
Only one input value will result in a true statement.

Answers

When both sides of the equation are simplified, the coefficients are the same.

Step-by-step explanation:

An equation has infinite solutions when both sides of the equation are simplified, the coefficients are the same

Answer:

The answer is A.

please help me i need this asap

Answers

Answer:

-2, -1

Step-by-step explanation:

B is the answer for you. Have a good day!

Other Questions
Consider these text features from Chapter 1 of Wheels of Change by Sue Macy. A high wheeler bicycle. In England, high wheelers were called penny farthings because the relationship of the front wheel to the back wheel resembled that of the large British penny coin to the smaller farthing (quarter penny) coin. How do these text features enhance the information provided in the book 2.Select the correct answer.Erica is working in the lab. She wants to remove the fine dust particles suspended in a sample of oil. Which method is she most likely to use? How many 20 can lengths of a pipe can be cut from a pipe 8.7 m long? Which phase of matter makes up stars?O liquidO gasO plasma Tess rolls 2 fair dice. What is the probability of obtaining two 4's? Determine the domain and range of the function f(x) = 23/1082O {x| all real numbers} {yly >0}O {x} all real numbers}; {yl y 2 0}O {x\ x >0}; {y| all real numbers}O {x\ x 2 0}; {yl all real numbers} If the balance of the cash account is $80,800 at the end of the month, what was the cash balance at the beginning of the month What dose that mean please help Assume that you have been hired by a large international bank that is looking to develop a smartphone app to help college students with their finances, credit scores, and investments. The new product development process for the app is just beginning, and you are looking at ways that social media usage can be improved during each step of the process. Whatever new product type a firm plans to develop, it will likely follow a formal new-product development (NPD) process. Organizations increasingly use social media to evaluate potential new products. Social media are especially valuable for small businesses and nonprofit organizations, which typically have less money to spend on the NPD process. Match each social media marketing action with the appropriate step of the new-product development process. The accumulation of glucose 6-phosphate inside a bacterial cell via phosphorylation of glucose is an example of: Solve the equation : 4x-(-2)=18 Consider the following hypothesis test: : : A sample of provided a sample mean and a sample standard deviation . Enter negative values as negative numbers. a. Compute the value of the test statistic (to three decimal places). What is the equation of the line written in general form? I need the answer ASAP What value of x is in the solution set of 2x 3 > 11 5x? -3 0 2 4 A skilled carpenter installed a roof on a new administrative building for a private not-for-profit free of charge. The not-for-profit would have had to pay $2,300 for this service if not donated. What entry should the not-for-profit make What is the domain of the square root function graphed below find the compound interest on rs 800 for 2 years at 8% per annum interest being payable annually 54% of U.S. adults have very little confidence in newspapers. You randomly select 10 U.S. adults. Find the probabilitythat the number of U.S. adults who have very little confidence in newspapers is (a) exactly five, (b) at least six, and (c)less than four.(a) P(5) =(Round to three decimal places as needed.)(b) P(x26) =(Round to three decimal places as needed.)(c) P(x Using a scale diagram, calculate the resultant force acting on a sailing boat when an easterly wind provides 2, point, 50, k, N,2.50kN of force, the tide provides 1, point, 20, k, N,1.20kN of force from the direction 30, point, 0, degrees,30.0 more northerly than the wind. Give your answer to 2 significant figures. Remember that 'an easterly wind' means a wind coming from the East does anyone know the domain and range of this?